LSAT and Law School Admissions Forum

Get expert LSAT preparation and law school admissions advice from PowerScore Test Preparation.

 Administrator
PowerScore Staff
  • PowerScore Staff
  • Posts: 8917
  • Joined: Feb 02, 2011
|
#41649
Complete Question Explanation
(The complete setup for this game can be found here: lsat/viewtopic.php?t=6962)

The correct answer choice is (D)

If F is not selected, then G must be selected, because F and G are the only two desserts (and we need at least one dessert). If G is selected, we also need to select O in compliance with the fourth rule:
PT65_D11 LG Explanations_game_#3_#15_diagram 1.png
With only one space remaining in the “unassigned” group, at least one of P or W must be selected. If so, then both P and W must be selected in accordance with the third rule:
PT65_D11 LG Explanations_game_#3_#15_diagram 2.png
Finally, recall that at least one hot food must always be selected. F, N, and T are the only hot foods, and so far we have none of them in our selection. So, we must include either N or T:
PT65_D11 LG Explanations_game_#3_#15_diagram 3.png
Judging from the language used in our answer choices, here it would be worth notating (using a subscript or a separate stack) the type of food each variable is:
PT65_D11 LG Explanations_game_#3_#15_diagram 4.png
Answer choice (A) is incorrect, because at least two main courses must be selected (P, O).

Answer choice (B) is incorrect, because W is a side dish that must be selected.

Answer choice (C) is incorrect, because only one hot food can selected (N or T).

Answer choice (D) is the correct answer choice, because all three main courses can be selected (N, O, P).

Answer choice (E) is incorrect, because at most two side dishes can be selected (V, a side dish, cannot be selected).
You do not have the required permissions to view the files attached to this post.
 lhm396
  • Posts: 1
  • Joined: May 14, 2014
|
#14693
This is the question about a luncheon...

#15 says if F is not selected, which one of the following could be true? The answer is exactly two hot foods are selected, but I don't understand how this answer is reached.

If F is not selected, then N and T are the only two hot foods left, but if N is selected, V is not selected along with F not being selected and since at least one dessert must be selected G must be selected since F and G are the only two desserts and if G is selected O must be selected which leaves my diagram

in -- N T G O _ | out -- F V _

but then P and W would have to be separated but the rules say if either P or W is selected they must both be selected so I am stuck on how having two hot foods (N and T since F is out in this question) could work while also satisfying the rule of at least one dessert, at least one main course and at least one side dish and keeping P and W together.

Thanks!
 Emily Haney-Caron
PowerScore Staff
  • PowerScore Staff
  • Posts: 577
  • Joined: Jan 12, 2012
|
#14709
Hi LHM,

The correct answer is actually D - "Exactly three main courses are selected." Your analysis regarding answer C is totally correct!

Emily

Get the most out of your LSAT Prep Plus subscription.

Analyze and track your performance with our Testing and Analytics Package.